Help with Epsilon Delta Proof of Multivariable Limit

In summary: In order to show this, you need to use the substitution x = a and y = -1 in the original inequality, and then use the Chain Rule.The Attempt at a SolutionLooking at the answer I see that the limit does not exist; however when I do the epsilon delta proof I can't see where I went wrong because I keep getting the result that it does :( ? So I attached a picture detailing my argument and I would love for someone to tell me where I went wrong. The following inequality from your picture does not hold if x2 + (y - 1)2 < 1 . In order to show this, you need to use the substitution x = a and y = -1 in the
  • #1
joe5185
10
1

Homework Statement


Hey guys. I am having a little trouble answering this question. I am teaching myself calc 3 and am a little confused here (and thus can't ask a teacher). I need to find the limit as (x,y) approaches (0,1) of f(x,y) when f(x,y)=(xy-x)/(x^2+y^2-2y+1).

Homework Equations


|f(x,y)-L|<epsilon
0<sqrt((x-a)^2+(y-b)^2))<delta
I made L=0, assumed epsilon>0

The Attempt at a Solution


Looking at the answer I see that the limit does not exist; however when I do the epsilon delta proof I can't see where I went wrong because I keep getting the result that it does :( ? So I attached a picture detailing my argument and I would love for someone to tell me where I went wrong. I chose L in the epsilon delta definition to be 0 because this is what I get when I approach (0,1) along x=0, y=1, and y=x^3+1 . I am aware that the limit does not exist because if you travel along x=y^2-1 you get a value other than zero. However my only concern is why my logic is not correct in the attached image. Thanks a lot! Also if you have tips for doing these epsilon delta proofs I would love to hear them.
 

Attachments

  • argument.png
    argument.png
    9 KB · Views: 2,251
Last edited:
Physics news on Phys.org
  • #2
You can factor the numerator and get x(y-1) . Meanwhile your denominator factors and you get x^2+(y-1)^2. If you let ## x=\epsilon ## (it approaches zero) and let ## y-1=\Delta ## you then get a simple expression for the limit in terms of ## \epsilon ## and ## \Delta ##. If you let ## \Delta=\alpha \epsilon ## the result depends on ## \alpha ##. Thereby you don't have a single limit that it converges to. And I see your error=your denominator is greater than zero but it is not greater than 1.(top line=your inequality is incorrect.)
 
  • Like
Likes joe5185
  • #3
thank you so much. I completely follow you here:). Do you mind elaborating on when I can use the technique where you choose what in your expression is delta and what is epsilon? It seems pretty powerful but I just want to make sure when and how to use it. Thanks for the help
 
  • #4
joe5185 said:
thank you so much. I completely follow you here:). Do you mind elaborating on when I can use the technique where you choose what in your expression is delta and what is epsilon? It seems pretty powerful but I just want to make sure when and how to use it. Thanks for the help
You have two variables, x, y that are approaching a,b respectively. Let ## x-a=\epsilon ## and ## y-b=\Delta ##. The ## \epsilon ## and the ## \Delta ## both approach zero, but there's nothing that says ## \epsilon=\Delta ##. You can let ## \Delta=\alpha \epsilon ##.The constant ## \alpha ## is quite arbitrary. If you could show your expression to give an answer that is independent of ## \alpha ##, then the limit would be what you computed by evaluating the expression with the ## \epsilon ## and ## \Delta ##. Hopefully this is helpful.
 
  • #5
very helpful! thanks
 
  • Like
Likes Charles Link
  • #6
joe5185 said:

Homework Statement


Hey guys. I am having a little trouble answering this question. I am teaching myself calc 3 and am a little confused here (and thus can't ask a teacher). I need to find the limit as (x,y) approaches (0,1) of f(x,y) when f(x,y)=(xy-x)/(x^2+y^2-2y+1).

Homework Equations


|f(x,y)-L|<epsilon
0<sqrt((x-a)^2+(y-b)^2))<delta
I made L=0, assumed epsilon>0

The Attempt at a Solution


Looking at the answer I see that the limit does not exist; however when I do the epsilon delta proof I can't see where I went wrong because I keep getting the result that it does :( ? So I attached a picture detailing my argument and I would love for someone to tell me where I went wrong.
...
The following inequality from your picture does not hold if x2 + (y - 1)2 < 1 .
upload_2016-6-19_18-53-34.png
 

What is an Epsilon Delta Proof of Multivariable Limit?

An Epsilon Delta Proof of Multivariable Limit is a mathematical method used to rigorously prove the existence of a limit for a function with multiple variables. It involves using the concepts of epsilon and delta to show that for any given small value of epsilon, there exists a corresponding small value of delta that ensures the function's output will be within the desired range.

How is an Epsilon Delta Proof of Multivariable Limit different from a single variable limit?

An Epsilon Delta Proof of Multivariable Limit is different from a single variable limit because it takes into account the behavior of a function with multiple variables, whereas a single variable limit only considers the behavior of a function with one variable. This makes an Epsilon Delta Proof more complex and requires additional steps and considerations.

What are the steps for completing an Epsilon Delta Proof of Multivariable Limit?

The steps for completing an Epsilon Delta Proof of Multivariable Limit include:

  1. Defining the limit and setting up the inequality for the desired range of the limit.
  2. Choosing a small value of epsilon and setting up the corresponding small value of delta.
  3. Manipulating the inequality to relate epsilon and delta.
  4. Using the given function and the value of delta to show that the output of the function will always be within the desired range of the limit.
  5. Concluding that the limit exists and is equal to the given value.

What are some common challenges when completing an Epsilon Delta Proof of Multivariable Limit?

Some common challenges when completing an Epsilon Delta Proof of Multivariable Limit include:

  • Choosing the appropriate value of delta that will guarantee the desired range of the limit.
  • Manipulating the inequality to relate epsilon and delta in a way that allows for the use of the given function.
  • Understanding the behavior of the function with multiple variables and how it affects the choice of epsilon and delta.

Why is an Epsilon Delta Proof of Multivariable Limit important in mathematics?

An Epsilon Delta Proof of Multivariable Limit is important in mathematics because it provides a rigorous and precise method for proving the existence of a limit for a function with multiple variables. This can be applied in various fields of mathematics and science, such as calculus, physics, and engineering, to ensure the accuracy and validity of mathematical models and calculations.

Similar threads

  • Calculus and Beyond Homework Help
Replies
9
Views
2K
  • Calculus and Beyond Homework Help
Replies
8
Views
1K
  • Calculus and Beyond Homework Help
Replies
19
Views
1K
  • Calculus and Beyond Homework Help
Replies
13
Views
489
  • Calculus and Beyond Homework Help
Replies
5
Views
878
  • Calculus and Beyond Homework Help
Replies
13
Views
2K
  • Calculus and Beyond Homework Help
Replies
3
Views
389
  • Calculus and Beyond Homework Help
Replies
13
Views
1K
  • Calculus and Beyond Homework Help
Replies
15
Views
4K
  • Calculus and Beyond Homework Help
Replies
12
Views
2K
Back
Top